Join our Telegram Channel

কষে দেখি 5.3 | 5. অনুপাত ও সমানুপাত | WBBSE Class 10 Math Solution

 5. অনুপাত ও সমানুপাত (Ratio and Proportion) || কষে দেখি 5.3 || WBBSE Class 10 Math Solution 


 কষে দেখি 5.3 সমাধান 


1. a:b=c:d হলে, দেখাই যে,
(i) \((a^2+b^2):(a^2-b^2)=(ac+bd):(ac-bd)\)
সমাধানঃ
ধরি, \(\frac{a}{b}=\frac{c}{d}=k\) (যেখানে k≠0)
∴ a=bk এবং c=dk

বামপক্ষ 

= \(\frac{a^2+b^2}{a^2-b^2}\)

= \(\frac{{(bk)}^2+b^2}{{(bk)}^2-b^2}\)

= \(\frac{{b^2k}^2+b^2}{{b^2k}^2-b^2}\)

= \(\frac{{b^2(k}^2+1)}{{b^2(k}^2-1)}\)

= \(\frac{{(k}^2+1)}{{(k}^2-1)}\)

ডানপক্ষ 
= \(\frac{ac+bd}{ac-bd}\)

= \(\frac{(bk)(dk)+bd}{(bk)(dk)-bd}\)

= \(\frac{{bdk}^2+bd}{{bdk}^2-bd}\)

= \(\frac{{bd(k}^2+1)}{{bd(k}^2-1)}\)

= \(\frac{{(k}^2+1)}{{(k}^2-1)}\)

বামপক্ষ = ডানপক্ষ [প্রমাণিত]


1. a:b=c:d হলে, দেখাই যে,
(ii) \((a^2+ab+b^2):(a^2-ab+b^2)\)
    \(=(c^2+cd+d^2):(c^2-cd+d^2)\)

সমাধানঃ
ধরি, \(\frac{a}{b}=\frac{c}{d}=k\) (যেখানে k≠0)
a=bk এবং c=dk

বামপক্ষ 

= \(\frac{a^2+ab+b^2}{a^2-ab+b^2}\)

= \(\frac{(b{k)}^2+bk.b+b^2}{(b{k)}^2+bk.b+b^2}\)

= \(\frac{b^2k^2+b^2.k+b^2}{b^2k^2-b^2.k+b^2}\)

= \(\frac{b^2{(k}^2+k+1)}{b^2{(k}^2-k+1)}\) 

= \(\frac{{(k}^2+k+1)}{{(k}^2-k+1)}\)

ডানপক্ষ 
= \(\frac{c^2+cd+d^2}{c^2-cd+d^2}\)

= \(\frac{(d{k)}^2+dk.b+d^2}{(d{k)}^2+dk.d+d^2}\)

= \(\frac{d^2k^2+d^2.k+d^2}{d^2k^2-d^2.k+d^2}\)

= \(\frac{d^2{(k}^2+k+1)}{d^2{(k}^2-k+1)}\) 

= \(\frac{{(k}^2+k+1)}{{(k}^2-k+1)}\)

বামপক্ষ = ডানপক্ষ [প্রমাণিত]


2. x:a=y:b=z:c হলে, প্রমাণ করি যে,
(i) \(\frac{x^3}{a^2}+\frac{y^3}{b^2}+\frac{z^3}{c^2}=\frac{{(x+y+z)}^3}{{(a+b+c)}^2}\)
সমাধানঃ

ধরি, \(\frac{x}{a}=\frac{y}{b}=\frac{z}{c}=k\) (যেখানে k≠0)
x=ak, y=bk এবং z=ck

বামপক্ষ \(= \frac{x^3}{a^2}+\frac{y^3}{b^2}+\frac{z^3}{c^2}\)

\(= \frac{{(ak)}^3}{a^2}+\frac{{(bk)}^3}{b^2}+\frac{{(ck)}^3}{c^2}\)

\(= \frac{{a^3k}^3}{a^2}+\frac{{b^3k}^3}{b^2}+\frac{{c^3k}^3}{c^2}\)

\(= {ak}^3+{bk}^3+{ck}^3 = k^3(a+b+c)\)

ডানপক্ষ \(= \frac{{(x+y+z)}^3}{{(a+b+c)}^2}\)

\(= \frac{{(ak+bk+ck)}^3}{{(a+b+c)}^2}\)

\(= \frac{{{k\left(a+b+c\right)}}^3}{{(a+b+c)}^2}\)

\(= \frac{{k^3\left(a+b+c\right)}^3}{{(a+b+c)}^2}= k^3(a+b+c)\)

∴ বামপক্ষ = ডানপক্ষ [প্রমাণিত]


2. x:a=y:b=z:c হলে, প্রমাণ করি যে,  
(ii) \(\frac{x^3+y^3+z^3}{a^3+b^3+c^3}=\frac{xyz}{abc}\)
সমাধানঃ
ধরি, \(\frac{x}{a}=\frac{y}{b}=\frac{z}{c}=k\) (যেখানে k≠0)
∴ x=ak, y=bk এবং z=ck

বামপক্ষ \(= \frac{x^3+y^3+z^3}{a^3+b^3+c^3}\)

    \(= \frac{{(ak)}^3+{(bk)}^3+{(ck)}^3}{a^3+b^3+c^3}\)

    \(= \frac{a^3k^3+b^3k^3+c^3k^3}{a^3+b^3+c^3}\)

            \(= \frac{{k^3(a}^3+b^3+c^3)}{a^3+b^3+c^3}\)

           \(= k^3\)

ডানপক্ষ \(= \frac{xyz}{abc}\)

\(= \frac{ak.bk.ck}{abc}\)

\(= k^3\)

বামপক্ষ = ডানপক্ষ [প্রমাণিত]


2. x:a=y:b=z:c হলে, প্রমাণ করি যে,
(iii) \((a^2+b^2+c^2)(x^2+y^2+z^2)=\left(ax+by+cz\right)^2\)
সমাধানঃ
ধরি, \(\frac{x}{a}=\frac{y}{b}=\frac{z}{c}=k\) (যেখানে k≠0)
x=ak, y=bk এবং z=ck

বামপক্ষ 

= \((a^2+b^2+c^2)(x^2+y^2+z^2)\)

= \((a^2+b^2+c^2)(a^2k^2+b^2k^2+c^2k^2)\)

= \(\left(a^2+b^2+c^2\right)k^2(a^2+b^2+c^2)\)

= \(k^2\left(a^2+b^2+c^2\right)^2\)

ডানপক্ষ  

= \(\left(ax+by+cz\right)^2\)

= \(\left(a.ak+b.bk+c.ck\right)^2\)

= \(\left\{k\left(a^2+b^2+c^2\right)\right\}^2\)

= \(k^2\left(a^2+b^2+c^2\right)^2\)

∴ বামপক্ষ = ডানপক্ষ [প্রমাণিত]


3. a:b=c:d=e:f হলে, প্রমাণ করি যে,
(i)প্রত্যেকটি অনুপাত = \(\frac{5a-7c-13e}{5b-7d-13f}\) 
সমাধানঃ
\(\frac{a}{b}=\frac{c}{d}=\frac{e}{f}\)
বা, \(\frac{5a}{5b}=\frac{7c}{7d}=\frac{13e}{13f}=\frac{5a-7c-13e}{5b-7d-13f}\)
[সংযোজন প্রক্রিয়ার সাহায্যে পাই]
প্রত্যেকটি অনুপাত = \(\frac{5a-7c-13e}{5b-7d-13f}\) [প্রমাণিত] 


3. a:b=c:d=e:f হলে, প্রমাণ করি যে,
(ii) \((a^2+c^2+e^2)(b^2+d^2+f^2)=\left(ab+cd+ef\right)^2\)
সমাধানঃ
ধরি, \(\frac{a}{b}=\frac{c}{d}=\frac{e}{f}=k\) (যেখানে k≠0)
a=bk, c=dk এবং e=fk

বামপক্ষ 

= \((a^2+c^2+e^2)(b^2+d^2+f^2)\)

= \((b^2k^2+d^2k^2+f^2k^2)(b^2+d^2+f^2)\)

= \(k^2(b^2+d^2+f^2)(b^2+d^2+f^2)\)

= \(k^2\left(b^2+d^2+f^2\right)^2\)

ডানপক্ষ 

= \(\left(ab+cd+ef\right)^2\)

= \(\left(bk.b+dk.d+fk.f\right)^2\)

= \(\left\{k\left(b^2+d^2+f^2\right)\right\}^2\)

= \(k^2\left(b^2+d^2+f^2\right)^2\)

∴ বামপক্ষ = ডানপক্ষ [প্রমাণিত]


4. যদি a:b=b:c হয়, তবে প্রমাণ করি যে,
(i) \(\left(\frac{a+b}{b+c}\right)^2=\frac{a^2+b^2}{b^2+c^2}\)
সমাধানঃ
ধরি, \(\frac{a}{b}=\frac{b}{c}=k\) (যেখানে k≠0)
a=bk এবং b=ck 
বা, \(a=ck.k=ck^2\)
বামপক্ষ = \(\left(\frac{a+b}{b+c}\right)^2\)
= \(\left(\frac{ck^2+ck}{ck+c}\right)^2\)
= \(\left\{\frac{ck(k+1)}{c(k+1)}\right\}^2= k^2\)
ডানপক্ষ = \(\frac{a^2+b^2}{b^2+c^2}\)
= \(\frac{c^2k^4+c^2k^2}{c^2k^2+c^2}\)
= \(\frac{c^2k^2(k^2+1)}{c^2(k^2+1)} = k^2\)
∴ বামপক্ষ = ডানপক্ষ [প্রমাণিত]


4. যদি a:b=b:c হয়, তবে প্রমাণ করি যে,
(ii) \(a^2b^2c^2\left(\frac{1}{a^3}+\frac{1}{b^3}+\frac{1}{c}\right)=a^3+b^3+c^3\)
সমাধানঃ
ধরি, \(\frac{a}{b}=\frac{b}{c}=k\) (যেখানে k≠0)
a=bk এবং b=ck 
বা, \(a=ck.k=ck^2\)

বামপক্ষ 

= \(a^2b^2c^2\left(\frac{1}{a^3}+\frac{1}{b^3}+\frac{1}{c^3}\right)\)

= \(c^2k^4{.c}^2k^2.c^2\left(\frac{1}{c^3k^6}+\frac{1}{c^3k^3}+\frac{1}{c^3}\right)\)

= \(c^6k^6\left(\frac{1+k^3+k^6}{c^3k^6}\right)\)

= \(c^3\left(1+k^3+k^6\right)\)

ডানপক্ষ 

= \(a^3+b^3+c^3\)

= \(c^3k^6+c^3k^3+c^3\)

= \(c^3\left(k^6+k^3+1\right)\)

= \(c^3\left(1+k^3+k^6\right)\)

∴ বামপক্ষ = ডানপক্ষ [প্রমাণিত]


4. যদি a:b=b:c হয়, তবে প্রমাণ করি যে,
(iii) \(\frac{abc{(a+b+c)}^3}{{(ab+bc+ca)}^3}=1\)
সমাধানঃ
ধরি, \(\frac{a}{b}=\frac{b}{c}=k\) (যেখানে k≠0)
a=bk এবং b=ck 
বা, \(a=ck.k=ck^2\)

\(\frac{abc{(a+b+c)}^3}{{(ab+bc+ca)}^3}\)

=\(\frac{ck^2.ck.c\left(ck^2+ck+c\right)^3}{{(ck^2.ck+ck.c+c.ck^2)}^3}\)

= \(\frac{c^3k^3\left\{c{(k}^2+k+1)\right\}^3}{{(c^2k^3+c^2k+c^2k^2)}^3}\)

= \(\frac{c^3k^3.c^3{{(k}^2+k+1)}^3}{\left\{c^2k(k^2+1+k)\right\}^3}\)

= \(\frac{c^6k^3{{(k}^2+k+1)}^3}{c^6k^3{{(k}^2+1+k)}^3}\)

= 1
\(\frac{abc{(a+b+c)}^3}{{(ab+bc+ca)}^3}=1\) [প্রমাণিত]


5. a, b, c, d ক্রমিক সমানুপাতী হলে, প্রমাণ করি যে,
(i) \((a^2+b^2+c^2)(b^2+c^2+d^2)\)
            \(=(ab+cd+cd)^2\)
সমাধানঃ
a, b, c, d ক্রমিক সমানুপাতী হলে \(\frac{a}{b}=\frac{b}{c}=\frac{c}{d}\)
ধরি, \(\frac{a}{b}=\frac{b}{c}=\frac{c}{d}=k\) (যেখানে, k≠0)
সুতরাং, a=bk, b=ck এবং c=dk
         \(b=dk×k= dk^2\)
এবং \(a=dk^2×k= dk^3\)  
বামপক্ষ 
\(=\left(a^2+b^2+c^2\right)\left(b^2+c^2+d^2\right)\)
\(=\left(d^2k^6+d^2k^4+d^2k^2\right)\left(d^2k^4+d^2k^2+d^2\right)\)
\(=d^2k^2\left(k^4+k^2+1\right)d^2(k^4+k^2+1)\)
\(=d^4k^2\left(k^4+k^2+1\right)^2\)

ডানপক্ষ
\(=\left(ab+bc+cd\right)^2\)
\(=\left(dk^3.\ dk^2+dk^2.dk+dk.d\right)^2\) 
\(=\left(d^2k^5+d^2k^3+d^2k\right)^2\)
\(=\left\{d^2k\left(k^4+k^2+1\right)^2\right\}^2\) 
\(=d^4k^2\left(k^4+k^2+1\right)^2\)

         বামপক্ষ = ডানপক্ষ [প্রমাণিত]

5. a, b, c, d ক্রমিক সমানুপাতী হলে, প্রমাণ করি যে,
(ii) \(\left(b-c\right)^2+\left(c-a\right)^2+\left(b-d\right)^2=\left(a-d\right)^2\)
সমাধানঃ
a, b, c, d ক্রমিক সমানুপাতী হলে \(\frac{a}{b}=\frac{b}{c}=\frac{c}{d}\)
ধরি, \(\frac{a}{b}=\frac{b}{c}=\frac{c}{d}=k\) (যেখানে, k≠0)
সুতরাং, a=bk, b=ck এবং c=dk
\(b=dk\times\ k=\ dk^2\)
এবং \(a=dk^2\times\ k=\ dk^3\)

বামপক্ষ
\(=\left(b-c\right)^2+\left(c-a\right)^2+\left(b-d\right)^2\)
\(=\left(dk^2-dk\right)^2+\left(dk-dk^3\right)^2+\left(dk^2-d\right)^2\)
\(=\left\{d\left(k^2-k\right)\right\}^2+\left\{d\left(k-k^3\right)\right\}^2+\left\{d\left(k^2-1\right)\right\}^2\)
\(=d^2\left\{\left(k^2-k\right)^2+\left(k-k^3\right)^2+\left(k^2-1\right)^2\right\}\)
\(=d^2\{k^4-2k^3+k^2+k^2-2k^4\)
                        \(+k^6+k^4-2k^2+1\}\)
\(=d^2{-2k^3+k^6+1}\)
\(=d^2{k^6-2k^3+1}\ =\ d^2\left(k^3-1\right)^2\)

ডানপক্ষ
\(=\left(a-d\right)^2\)
\(=\left(dk^3-d\right)^2\)
\(=\left\{d\left(k^3-1\right)\right\}^2\)
\(=d^2\left(k^3-1\right)^2\)

         বামপক্ষ = ডানপক্ষ [প্রমাণিত]


6. (i) যদি \(\frac{m}{a}=\frac{n}{b}\) হয়, তবে দেখাই যে, 
\((m^2+n^2)(a^2+b^2)=(am+bn)^2\)
সমাধানঃ
ধরি, \(\frac{m}{a}=\frac{n}{b}=k\) (যেখানে, k≠0)
সুতরাং, m=ak এবং n=bk
বামপক্ষ
\(=(a^2.k^2+\ b^2.k^2)(a^2+b^2)\)
\(=k^2(a^2+b^2)(a^2+b^2)\)
\(=k^2\left(a^2+b^2\right)^2\)

ডানপক্ষ
\(=\left(am+bn\right)^2\)
\(=\left(a.ak+b.bk\right)^2\)
\(=\left(a^2k+b^2k\right)^2\)
\(=\left\{k\left(a^2+b^2\right)\right\}^2\)
\(=k^2\left(a^2+b^2\right)^2\)

    বামপক্ষ = ডানপক্ষ [প্রমাণিত]


6. (ii) যদি \(\frac{a}{b}=\frac{x}{y}\) হয়, তবে দেখাই যে, 
\(\left(a+b\right)\left(a^2+b^2\right)x^3=\left(x+y\right)\left(x^2+y^2\right)a^3\)
সমাধানঃ
ধরি,  (যেখানে, k≠0)
সুতরাং, a=bk এবং x=yk

বামপক্ষ
\(=\left(a+b\right)\left(a^2+b^2\right)x^3\)
\(=\left(bk+b\right)\left(b^2k^2+b^2\right)y^3k^3\)
\(=b\left(k+1\right)b^2\left(k^2+1\right)y^3k^3\) 
\(=b^3y^3k^3(k+1)(k^2+1)\) 

ডানপক্ষ
\(=\left(x+y\right)\left(x^2+y^2\right)a^3\)
\(=\left(yk+y\right)\left(\ y^2k^2+y^2\right)b^3k^2\)
\(=y(k+1).\ y^2\left(k^2+1\right)b^3k^3\)
\(=b^3y^3k^3(k+1)(k^2+1)\)

    বামপক্ষ = ডানপক্ষ [প্রমাণিত]


6. (iii) যদি \(\frac{x}{lm-n^2}=\frac{y}{mn-l^2}=\frac{z}{nl-m^2}\) হয়, 
তবে দেখাই যে, \(lx+my+nz=0\)
সমাধানঃ
ধরি, \(\frac{x}{lm-n^2}=\frac{y}{mn-l^2}=\frac{z}{nl-m^2}=k\) (যেখানে, k≠0)
সুতরাং, \(x=k(lm-n^2)\)
\(y=k(mn-l^2)\)
এবং \(z=k(nl-m^2)\)
\(lx+my+nz\)  
\(=l.\ k\left(lm-n^2\right)+m.k\left(mn-l^2\right)\)
            \(+n.k(nl-m^2)\)
\( =k\left(l^2m-n^2l+m^2n-l^2m+n^2l-m^2n\right)\)
= k×0
= 0
∴ \(lx+my+nz=0\) [প্রমাণিত]


6. (iv) \(\frac{x}{b+c-a}=\frac{y}{c+a-b}=\frac{z}{a+b-c}\) হলে, 
দেখাই যে, (b-c)x+(c-a)y+(a-b)z=0
সমাধানঃ
ধরি, \(\frac{x}{b+c-a}=\frac{y}{c+a-b}=\frac{z}{a+b-c}=k\) (যেখানে, k≠0)
সুতরাং, x=k(b+c-a), y=k(c+a-b) এবং z=k(a+b-c)
(b-c)x+(c-a)y+(a-b)z
= (b-c)(b+c-a)+(c-a)(c+a-b)+(a-b)(a+b-c)
= (b-c)(b+c)-a(b-c)+(c-a)(c+a)-b(c-a)+(a-b)(a+b)-c(a-b)
= \(b^2-c^2-ab+ac+c^2-a^2\)
            \(-bc+ab-a^2-b^2-ac+bc\)
= 0
∴ (b-c)x+(c-a)y+(a-b)z=0 [প্রমাণিত]


6. (v) \(\frac{x}{y}=\frac{a+2}{a-2}\) হলে, 
দেখাই যে, \(\frac{x^2-y^2}{x^2+y^2}=\frac{4a}{a^2+4}\)
সমাধানঃ
\(\frac{x}{y}=\frac{a+2}{a-2}\)
বা, \(\frac{x^2}{y^2}=\frac{\left(a+2\right)^2}{\left(a-2\right)^2}\) [উভয়পক্ষকে বর্গ করে পাই]
বা, \(\frac{x^2+y^2}{x^2-y^2}=\frac{\left(a+2\right)^2+\left(a-2\right)^2}{\left(a+2\right)^2-\left(a-2\right)^2}\)
                            [যোগ-ভাগ প্রক্রিয়ার সাহয্যে পাই]
বা, \(\frac{x^2+y^2}{x^2-y^2}=\frac{4.a.2}{2(a^2+2^2)}\)
\(\frac{x^2-y^2}{x^2+y^2}=\frac{4a}{a^2+4}\) [প্রমাণিত]


6. (vi) \(x=\frac{8ab}{a+b}\) হলে, 
\(\left(\frac{x+4a}{x-4a}+\frac{x+4b}{x-4b}\right)\)-এর মান হিসাব করে লিখি। 
সমাধানঃ
\(x=\frac{8ab}{a+b}\)

বা, \(\frac{x}{4a}=\frac{2b}{a+b}\) [উভয়পক্ষকে 4a দ্বারা ভাগ করে পাই]

বা, \(\frac{x+4a}{x-4a}=\frac{2b+a+b}{2b-(a+b)}\) [যোগ-ভাগ প্রক্রিয়ার সাহায্যে পাই]

বা, \(\frac{x+4a}{x-4a}=\frac{3b+a}{b-a}\)

আবার, \(x=\frac{8ab}{a+b}\)

বা, \(\frac{x}{4b}=\frac{2a}{a+b}\) [উভয়পক্ষকে 4b দ্বারা ভাগ করে পাই]

বা, \(\frac{x+4b}{x-4b}=\frac{2a+a+b}{2a-(a+b)}\) [যোগ-ভাগ প্রক্রিয়ার সাহায্যে পাই]

বা, \(\frac{x+4b}{x-4b}=\frac{3a+b}{a-b}\)

∴ \(\frac{x+4a}{x-4a}+\frac{x+4b}{x-4b}\)

\(=\frac{3b+a}{b-a}+\frac{3a+b}{a-b}\)

\(= \frac{3b+a}{b-a}-\frac{3a+b}{b-a}\)

\(= \frac{3b+a-3a-b}{b-a}\)

\(= \frac{2(b-a)}{b-a}= 2\)


7. (i) \(\frac{a}{3}=\frac{b}{4}=\frac{c}{7}\) হলে, 
দেখাই যে, \(\frac{a+b+c}{c}=2\)
সমাধানঃ
ধরি, \(\frac{a}{3}=\frac{b}{4}=\frac{c}{7}=k\) (যেখানে, k≠0)
সুতরাং, a=3k, b=4k এবং c=7k

\(\frac{a+b+c}{c}\)

=\(\frac{3k+4k+7k}{7k}\)

= \(\frac{14k}{7k} = 2\)

∴ \(\frac{a+b+c}{c}=2\) [প্রমাণিত]


7. (ii) \(\frac{a}{q-r}=\frac{b}{r-p}=\frac{c}{p-q}\) হলে, 
দেখাই যে, a+b+c=0=pa+qb+rc
সমাধানঃ
ধরি, \(\frac{a}{q-r}=\frac{b}{r-p}=\frac{c}{p-q}=k\) (যেখানে k≠0)
সুতরাং, a=k(q-r), b=k(r-p) এবং c=k(p-q)

a+b+c
= k(q-r)+k(r-p)+k(p-q)
= k(q-r+r-p+p-q)
= k×0
= 0

pa+qb+rc
= pk(q-r)+qk(r-p)+rk(p-q)
= pqk-prk+qrk-pqk+prk-qrk
= 0


7. (iii) \(\frac{ax+by}{a}=\frac{bx-ay}{b}\) হলে, 
দেখাই যে প্রতিটি অনুপাত x-এর সমান। 
সমাধানঃ

\(\frac{ax+by}{a}=\frac{bx-ay}{b}\)

বা, \(\frac{a\left(ax+by\right)}{a.a}=\frac{b\left(bx-ay\right)}{b.b}\)

= \(\frac{a\left(ax+by\right)+b(bx-ay)}{a^2+b^2}\) [সংযোজন প্রক্রিয়ার সাহায্যে পাই]

= \(\frac{b^2x+aby+a^2x-aby}{a^2+b^2}\)

= \(\frac{x(a^2+b^2)}{a^2+b^2} = x\)

    প্রতিটি অনুপাত x-এর সমান [প্রমাণিত]


8. (i) যদি \(\frac{a+b}{b+c}=\frac{c+d}{d+a}\) হয়, 
তবে প্রমাণ করি যে, c=a অথবা a+b+c+d=0
সমাধানঃ
\(\frac{a+b}{b+c}=\frac{c+d}{d+a}\)
বা, (a+b)(d+a)=(c+d)(b+c)
বা, \(ad+bd+a^2+ab=bc+bd+c^2+cd\)
বা, \(ad+bd+a^2+ab-bc-bd-c^2-cd=0\)
বা, \(a^2-c^2+ad-cd+ab-bc=0\)
বা, (a+c)(a-c)+d(a-c)+b(a-c)=0
বা, (a-c)(a+c+d+b)=0
হয়, a-c=0 অথবা, a+c+d+b=0
বা, c=a বা, a+b+c+d=0
∴ হয়, c=a এবং a+b+c+d=0 [প্রমাণিত]


8.(ii) যদি \(\frac{x}{b+c}=\frac{y}{c+a}=\frac{z}{a+b}\) হয়, 
দেখাই যে, \(\frac{a}{y+z-x}=\frac{b}{z+x-y}=\frac{c}{x+y-z}\)
সমাধানঃ
ধরি, \(\frac{x}{b+c}=\frac{y}{c+a}=\frac{z}{a+b}=k\) (যেখানে, k≠0)
সুতরাং, x=k(b+c), y=k(c+a) এবং z=k(a+b)

\(\frac{a}{y+z-x}\)

\(=\frac{a}{k\left(c+a\right)+k\left(a+b\right)-k(b+c)}\)

=\(\frac{a}{ck+ak+ak+bk-bk-ck}\)

=\(\frac{a}{2ak} = \frac{1}{2k}\)

 \(\frac{b}{z+x-y}\)

\(=\frac{b}{k\left(a+b\right)+k\left(b+c\right)-k\left(c+a\right)}\)

=\(\frac{b}{ak+bk+bk+ck-ck-ak}\)

=\(\frac{b}{2bk} = \frac{1}{2k}\)

 \(\frac{c}{x+y-z}\)

\(=\frac{c}{k\left(b+c\right)+k\left(c+a\right)-k(a+b)}\)

=\(\frac{c}{bk+ck+ck+ak-ak-bk}\)

=\(\frac{c}{2ck} = \frac{1}{2k}\)

    \(\frac{a}{y+z-x}=\frac{b}{z+x-y}=\frac{c}{x+y-z}\) [প্রমাণিত]


8. (iii) \(\frac{x+y}{3a-b}=\frac{y+z}{3b-c}=\frac{z+x}{3c-a}\) হলে, 
দেখাই যে, \(\frac{x+y+z}{a+b+c}=\frac{ax+by+cz}{a^2+b^2+c^2}\) 
সমাধানঃ 

\(\frac{x+y}{3a-b}\)

\(=\frac{y+z}{3b-c}=\frac{z+x}{3c-a}\) 

= \(\frac{x+y+y+z+z+x}{3a-b+3b-c+3c-a}\) [সংযোজন প্রক্রিয়ার সাহায্যে পাই] 

= \(\frac{2(x+y+z)}{2(a+b+c)}\) 

= \(\frac{x+y+z}{a+b+c}\) 


\(\frac{x+y+z}{a+b+c}=\frac{x+y}{3a-b}\) 

=\(\frac{x+y+z-x-y}{a+b+c-3a+b}\) [সংযোজন প্রক্রিয়ার সাহায্যে পাই] 

=\(\frac{z}{2b-2a+c}\) 

 একইরকম ভাবে পাই, 

 \(\frac{x+y+z}{a+b+c}=\frac{y+z}{3b-c}=\ \frac{x+y+z-y-z}{a+b+c-3b+c}=\ \frac{x}{a-2b+2c}\)

 এবং \(\frac{x+y+z}{a+b+c}=\frac{z+x}{3c-a}= \frac{x+y+z-z-x}{a+b+c-3c+a}= \frac{y}{2a+b-2c}\)

 ∴\(\frac{x}{a-2b+2c}=\frac{y}{2a+b-2c}=\frac{z}{2b-2a+c}\)

 = \(\frac{ax+by+cz}{a\left(a-2b+2c\right)+b\left(2a+b-2c\right)+c(2b-2a+c)}\) 
                    [সংযোজন প্রক্রিয়ার সাহায্যে পাই] 

= \(\frac{ax+by+cz}{a^2-2ab+2ac+2ab+b^2-2bc+2bc-2ac+c^2}\) 

 =\(\frac{ax+by+cz}{a^2+b^2+c^2}\)

 ∴ \(\frac{x+y+z}{a+b+c}=\frac{ax+by+cz}{a^2+b^2+c^2}\) [প্রমাণিত] 

8. (iv) \(\frac{x}{a}=\frac{y}{b}=\frac{z}{c}\) হলে, 
দেখাই যে, \(\frac{x^2-yz}{a^2-bc}=\frac{y^2-zx}{b^2-ca}=\frac{z^2-xy}{c^2-ab}\)
সমাধানঃ
ধরি, \(\frac{x}{a}=\frac{y}{b}=\frac{z}{c}=k\) (যেখানে k≠0)
সুতরাং, x=ak, y=bk এবং z=ck

\(\frac{x^2-yz}{a^2-bc}=\frac{a^2k^2-bk.ck}{a^2-bc}=\frac{k^2(a^2-bc)}{(a^2-bc)}=k^2\)

\(\frac{y^2-zx}{b^2-ca}=\frac{b^2k^2-ck.ak}{b^2-ca}=\frac{k^2(b^2-ca)}{(b^2-ca)}=k^2\)

\(\frac{z^2-xy}{c^2-ab}=\frac{c^2k^2-ak.bk}{c^2-ab}=\frac{k^2(c^2-ab)}{(c^2-ab)}=k^2\)

∴ \(\frac{x^2-yz}{a^2-bc}=\frac{y^2-zx}{b^2-ca}=\frac{z^2-xy}{c^2-ab}\) [প্রমাণিত]


9. (i) যদি \(\frac{3x+4y}{3u+4v}=\frac{3x-4y}{3u-4v}\) হয়, 
তবে দেখাই যে \(\frac{x}{y}=\frac{u}{v}\)
সমাধানঃ

\(\frac{3x+4y}{3u+4v}=\frac{3x-4y}{3u-4v}\)

বা, \(\frac{3x+4y}{3x-4y}=\frac{3u+4v}{3u-4v}\)

বা, \(\frac{(3x+4y)+(3x-4y)}{\left(3x+4y\right)-(3x-4y)}=\frac{\left(3u+4v\right)-(3u-4v)}{\left(3u+4v\right)+(3u-4v)}\)
[যোগ-ভাগ প্রক্রিয়ার সাহায্যে পাই]

বা, \(\frac{3x+4y+3x-4y}{3x+4y-3x+4y}=\frac{3u+4v+3u-4v}{3u+4v-3u+4v}\)

বা, \(\frac{6x}{8y}=\frac{6u}{8v}\)

∴ \(\frac{x}{y}=\frac{u}{v}\) [প্রমাণিত]


9. (ii) (a+b+c+d):(a+b-c-d)=(a-b+c-d):(a-b-c+d)
হলে, প্রমাণ করি যে, a:b=c:d
সমাধানঃ
\(\frac{(a+b+c+d)}{(a+b-c-d)}=\frac{(a-b+c-d)}{(a-b-c+d)}\)

বা, \(\frac{(a+b+c+d)+(a+b-c-d)}{(a+b+c+d)-(a+b-c-d)}=\frac{(a-b+c-d)+(a-b-c+d)}{(a-b+c-d)-(a-b-c+d)}\)
[যোগ-ভাগ প্রক্রিয়ার সাহায্যে পাই]

বা, \(\frac{a+b+c+d+a+b-c-d}{a+b+c+d-a-b+c+d)}=\frac{a-b+c-d+a-b-c+d}{a-b+c-d-a+b+c-d}\)

বা, \(\frac{2(a+b)}{2(c+d)}=\frac{2(a-b)}{2(c-d)}\)

বা, \(\frac{(a+b)}{(c+d)}=\frac{(a-b)}{(c-d)}\)

বা, \(\frac{(a+b)}{(a-b)}=\frac{(c+d)}{(c-d)}\)

বা, \(\frac{\left(a+b\right)+(a-b)}{\left(a+b\right)-(a-b)}=\frac{\left(c+d\right)+(c-d)}{\left(c+d\right)-(c-d)}\) 
[যোগ-ভাগ প্রক্রিয়ার সাহায্যে পাই]

বা, \(\frac{a+b+a-b}{a+b-a+b}=\frac{c+d+c-d}{c+d-c+d}\)

বা, \(\frac{2a}{2b}=\frac{2c}{2d}\)

বা, \(\frac{a}{b}=\frac{c}{d}\)

a:b=c:d [প্রমাণিত]


10. (i) \(\frac{a^2}{b+c}=\frac{b^2}{c+a}=\frac{c^2}{a+b}=1\) হলে, 
দেখাই যে, \(\frac{1}{1+a}+\frac{1}{1+b}+\frac{1}{1+c}=1\)
সমাধানঃ
 \(\frac{a^2}{b+c}=\frac{b^2}{c+a}=\frac{c^2}{a+b}=1\)
সুতরাং, \(a^2=b+c, b^2=c+a\) 
এবং \(c^2=a+b\)

\(\frac{1}{1+a}+\frac{1}{1+b}+\frac{1}{1+c}\)
=\(\frac{a}{a+a^2}+\frac{b}{b+b^2}+\frac{c}{c+c^2}\)
= \(\frac{a}{a+b+c}+\frac{b}{b+c+a}+\frac{c}{c+a+b}\)
=  \(\frac{a+b+c}{a+b+c}\)
= 1

∴ \(\frac{1}{1+a}+\frac{1}{1+b}+\frac{1}{1+c}=1\) [প্রমাণিত]


10. (ii) \(x^2:(by+cz)=y^2:(cz+ax)=z^2:(ax+by)=1\) হলে,
দেখাই যে, \(\frac{a}{a+x}+\frac{b}{b+y}+\frac{c}{c+z}=1\)
সমাধানঃ
\(\frac{x^2}{by+cz}=\frac{y^2}{cz+ax}=\frac{z^2}{ax+by}=1\)
সুতরাং, 
\(y^2=cz+ax\)
\(y^2=cz+ax\)
এবং \(z^2=ax+by\)

 \(\frac{a}{a+x}+\frac{b}{b+y}+\frac{c}{c+z}\)
 \(=\frac{ax}{ax+x^2}+\frac{by}{by+y^2}+\frac{cz}{cz+z^2}\)
 \(=\frac{ax}{ax+by+cz}+\frac{by}{by+cz+bx}+\frac{cz}{cz+ax+by}\)
 \(=\frac{ax+by+cz}{ax+by+cz}\)
= 1

∴ \(\frac{a}{a+x}+\frac{b}{b+y}+\frac{c}{c+z}=1\)  [প্রমাণিত]


11. (i) \(\frac{x}{xa+yb+zc}=\frac{y}{ya+zb+xc}=\frac{z}{za+xb+yc}\) এবং x+y+z≠0 হলে, 
দেখাই যে, প্রতিটি অনুপাত \(\frac{1}{a+b+c}\) -এর সমান
সমাধানঃ

\(\frac{x}{xa+yb+zc}=\frac{y}{ya+zb+xc}=\frac{z}{za+xb+yc}\) 

\(= \frac{x+y+z}{xa+yb+zc+ya+zb+xc+za+xb+yc}\)
[সংযোজন প্রক্রিয়ার সাহায্যে পাই]

\(= \frac{x+y+z}{xa+xb+xc+ya+yb+yc+za+zb+zc}\)

\(= \frac{x+y+z}{x\left(a+b+c\right)+y(a+b+c)+z(a+b+c)}\)

\(= \frac{x+y+z}{(a+b+c)(x+y+z)}\)

\(= \frac{1}{a+b+c}\)

প্রতিটি অনুপাত \(\frac{1}{a+b+c}\) -এর সমান [প্রমাণিত]


11. (ii) \(\frac{x^2-yz}{a}=\frac{y^2-zx}{b}=\frac{z^2-xy}{c}\) হলে, 
প্রমাণ করি যে, (a+b+c)(x+y+z)=ax+by+cz
সমাধানঃ
 \(\frac{x^2-yz}{a}=\frac{y^2-zx}{b}=\frac{z^2-xy}{c}\)          

বা, \(\frac{a}{x^2-yz}=\frac{b}{y^2-zx}=\frac{c}{z^2-xy}=k\) ধরি, (যেখানে, k≠0)                 

সুতরাং, \(a=k(x^2-yz)\)
\(b=k(y^2-zx)\) এবং \(c=k(z^2-xy)\)

বামপক্ষ 
= (a+b+c)(x+y+z)
\(= {k\left(x^2-yz\right)+k\left(y^2-zx\right)+k(z^2-xy)}(x+y+z)\)
\(= k\left(x^2-yz+y^2-zx+z^2-xy\right)(x+y+z)\)
\(= k.(x+y+z)(x^2+y^2+z^2-xy-yz-zx)\) 
\(=\ k.(x^3+y^3+z^3-3xyz)\)

ডানপক্ষ 
= ax+by+cz 
\(= k\left(x^2-yz\right).x+k\left(y^2-zx\right).y+k(z^2-xy).z\)
\(= k(x^3-xyz+y^3-xyz+z^3-xyz)\)
\(= k(x^3+y^3+z^3-3xyz)\)
বামপক্ষ = ডানপক্ষ [প্রমাণিত]


12. অতিসংক্ষিপ্ত উত্তরধর্মী প্রশ্ন(V.S.A.)
(A) বহুবিকল্পীয় প্রশ্ন (M.C.Q):  
(i) 3, 4 এবং 6-এর চতুর্থ সমানুপাতী
(a) 8 (b) 10 (c) 12 (d) 24
সমাধানঃ
ধরি, 3, 4 এবং 6-এর চতুর্থ সমানুপাতী x
\(\frac{3}{4}=\frac{6}{x}\)
\(x=\frac{6\times4}{3} = 8\)
উত্তরঃ (a) 8


(ii) 8 এবং 12-এর তৃতীয় সমানুপাতী
(a) 12 (b) 16 (c) 18 (d) 20
সমাধানঃ
ধরি, 8 এবং 12-এর তৃতীয় সমানুপাতী x
\(\frac{8}{12}=\frac{12}{x}\)
\(x=\frac{12\times12}{8} = 18\)
উত্তরঃ (c) 18


(iii) 16 এবং 25-এর মধ্য সমানুপাতী
(a) 400 (b) 100 (c) 20 (d) 40
সমাধানঃ
ধরি, 16 এবং 25-এর মধ্য সমানুপাতীx
\(\frac{16}{x}=\frac{x}{25}\)
বা, \(x^2=400\)
∴ \(x=\sqrt{400} = 20\)
উত্তরঃ (c) 20


(iv) a একটি ধনাত্মক সংখ্যা এবং \(a:\frac{27}{64}=\frac{3}{4}:a\) হলে,
a –এর মান
(a) \(\frac{81}{256}\) (b) 9 (c) \(\frac{9}{16}\) (d) \(\frac{16}{9}\)
সমাধানঃ
\(a:\frac{27}{64}=\frac{3}{4}:a\)
বা, \(\frac{a}{\frac{27}{64}}=\frac{\frac{3}{4}}{a}\)
বা, \(a^2=\frac{3}{4}\times\frac{27}{64}\)
বা, \(a^2=\frac{81}{256}\)
∴ \(a=\sqrt{\frac{81}{256}} = \frac{9}{16}\)
উত্তরঃ (c) \(\frac{9}{16}\)


(v) 2a=3b=4c হলে, a:b:c হবে
(a) 3:4:6 (b) 4:3:6 (c) 3:6:4 (d) 6:4:3
সমাধানঃ
ধরি, 2a=3b=4c=k (যেখানে, k≠0)
সুতরাং, \(a=\frac{k}{2}, b=\frac{k}{3}\)
এবং \(c=\frac{k}{4}\)
∴a:b:c 
= \(\frac{k}{2}:\frac{k}{3}:\frac{k}{4}\)
= \(\frac{12k}{2}:\frac{12k}{3}:\frac{12k}{4}\)
= 6:4:3
উত্তরঃ (d) 6:4:3


(B) নীচের বিবৃতিগুলি সত্য না মিথ্যা লিখিঃ
(i) ab:c², bc:a² এবং ca:b² –এর যৌগিক অনুপাত 1:1
সমাধানঃ
ab:c², bc:a² এবং ca:b² –এর 
যৌগিক অনুপাত = ab×bc×ca:c²×a²×b²
= a²b²c²: a²b²c²
= 1:1
উত্তরঃ সত্য


(ii) \(x^3y, x^2y^2\) এবং \(xy^3\) ক্রমিক সমানুপাতী
সমাধানঃ
\(\frac{x^3y}{x^2y^2} = \frac{x}{y}\)
এবং \(\frac{x^2y^2}{xy^3} = \frac{x}{y}\)
\(\frac{x^3y}{x^2y^2}=\frac{x^2y^2}{xy^3}\)
\(x^3y, x^2y^2\) এবং \(xy^3\) ক্রমিক সমানুপাতী 
উত্তরঃ সত্য 


(C) শূণ্যস্থান পূরণ করিঃ
(i) তিনটি ক্রমিক সমানুপাতী ধনাত্মক সংখ্যার গুনফল 64 হলে, তাদের মধ্যসমানুপাতী ________
সমাধানঃ
ধরি, তিনটি ক্রমিক সমানুপাতী সংখ্যা হল a, b ও c
abc=64
যেহেতু, সংখ্যা তিনটি ক্রমিক সমানুপাতী
\(\frac{a}{b}=\frac{b}{c}\)
বা, \(b^2=ac\)
বা, \(b^2=\frac{64}{b}\)
বা, \(b^3=64\)
বা, \(b^3=43\)
∴ b=4
∴ তিনটি ক্রমিক সমানুপাতী ধনাত্মক সংখ্যার গুনফল 64 হলে, তাদের মধ্যসমানুপাতী 4
উত্তরঃ 4


(ii) a:2=b:5=c:8 হলে, a-এর 50%=b-এর20%=c-এর_____%
সমাধানঃ
a:2=b:5=c:8 
∴ \(\frac{a}{2}=\frac{b}{5}=\frac{c}{8}\)
বা, \(a\times\frac{50}{100}=b\times\frac{20}{100}=c\times\frac{\frac{25}{2}}{100}\)
a-এর 50%=b-এর20%=c-এর \(\frac{25}{2}\ %\)
বা, a-এর 50%=b-এর20%=c-এর \(12\frac{1}{2}\ %\)
উত্তরঃ \(12\frac{1}{2}\)


(iii) (x+2) এবং (x-3) এর মধ্য সমানুপাতী x হলে, 
x এর মান _____
সমাধানঃ
(x+2) এবং (x-3) এর মধ্য সমানুপাতী x
\(\frac{x+2}{x}=\frac{x}{x-3}\)
বা, \(\left(x+2\right)\left(x-3\right)=x^2\)
বা, \(x^2+2x-3x-6-x^2=0\)
বা, -x-6=0
বা, x+6=0
∴ x=-6
উত্তরঃ -6


13.  সংক্ষিপ্ত উত্তরধর্মী প্রশ্ন (S.A.)
(i) \(\frac{a}{2}=\frac{b}{3}=\frac{c}{4}=\frac{2a-3b+4c}{p}\) হলে, p-এর মান নির্ণয় করি।
সমাধানঃ
ধরি, \(\frac{a}{2}=\frac{b}{3}=\frac{c}{4}=\frac{2a-3b+4c}{p}=k\) (যেখানে, k≠0)
সুতরাং, a=2k, b=3k, c=4k
এবং 2a-3b+4c=kp 
বা, 2.2k-3.3k+4.4k=pk
বা, 4k-9k+16k=pk
বা, 11k=pk
p=11
∴ p এর মান 11


 (ii) \(\frac{3x-5y}{3x+5y}=\frac{1}{2}\) হলে, \(\frac{3x^2-5y^2}{3x^2+5y^2}\) –এর মান নির্ণয় করি।
সমাধানঃ
\(\frac{3x-5y}{3x+5y}=\frac{1}{2}\)
বা, 6x-10y=3x+5y
বা, 6x-3x=5y+10y
বা, 3x=15y
x=5y
\(\frac{3x^2-5y^2}{3x^2+5y^2}\)
\(=\frac{3{.25y}^2-5y^2}{3.25y^2+5y^2}\)
\(=\ \frac{{75y}^2-5y^2}{{75y}^2+5y^2}\)
\(= \frac{{70y}^2}{{80y}^2}=\frac{7}{8}\)


(iii) a:b=3:4 এবং x:y=5:7 হলে, (3ax-by):(4by-7ax) কত নির্ণয় করি।
সমাধানঃ
ধরি, a=3k, b=4k এবং x=5p এবং y=7p
\(\frac{3ax-by}{4by-7ax}=\frac{3.3k.5p-4k.7p}{4.4k.7p-7.3k.5p}\)
\(= \frac{45kp-28kp}{112kp-105kp}\)
\(= \frac{17kp}{7kp}=\frac{17}{7}\)
∴ (3ax-by):(4by-7ax) = 17:7


(iv) x, 12, y, 27 ক্রমিক সমানুপাতী হলে, x ও y এর ধনাত্মক মান নির্ণয় করি।
সমাধানঃ
x, 12, y, 27 ক্রমিক সমানুপাতী
\(\frac{x}{12}=\frac{12}{y}=\frac{y}{27}\)
দ্বিতীয় ও তৃতীয় অনুপাত থেকে পাই,
\(\frac{12}{y}=\frac{y}{27}\)
বা, \(y^2=324\)
∴ \(y=\sqrt{324}=18\) [∵ y এর মান ধনাত্মক]
প্রথম ও দ্বিতীয় অনুপাত থেকে পাই,
\(\frac{x}{12}=\frac{12}{y}\)
বা, \(\frac{x}{12}=\frac{12}{18}\)
∴ \(x=\frac{12\times12}{18}=8\) [∵ x এর মান ধনাত্মক]
∴ x এর ধনাত্মক মান 8 এবং y এর ধনাত্মক মান 18


(v) a:b=3:2 এবং b:c=3:2 হলে, a+b:b+c কত নির্ণয় করি।
সমাধানঃ
\(\frac{a}{b}=\frac{3}{2}=\frac{3\times3}{2\times3}=\frac{9}{6}\)
এবং \(\frac{b}{c}=\frac{3}{2}=\frac{3\times2}{2\times2}=\frac{6}{4}\)
∴ a:b:c=9:6:4
ধরি, a=9k, b=6k এবং c=4k
∴ a+b:b+c=(9k+6k):(6k+4k)
= 15k:10k 
        = 15:10
        = 3:2




Post a Comment

0 Comments